LSAT and Law School Admissions Forum

Get expert LSAT preparation and law school admissions advice from PowerScore Test Preparation.

 Administrator
PowerScore Staff
  • PowerScore Staff
  • Posts: 8917
  • Joined: Feb 02, 2011
|
#23049
Complete Question Explanation

Justify the Conclusion. The correct answer choice is (D)

Vanwilligan argues that the salaries received by professional athletes are fair because they make enormous profits for their teams' owners, which in turn justifies the owners' willingness to pay them. Even though his argument is an economic one, the conclusion takes a more ethical, or moral stance: instead of saying simply that athletes' salaries are justified by the market conditions in which they are determined, Vanwilligan claims that they are fair.

You should immediately notice the logical gap between an employer's willingness to pay a high salary and that salary's inherent fairness. If the fairness of an athlete's compensation is determined by factors other than employers' willingness to pay for it, the argument would fall apart. To prove Vanwilligan's conclusion, you should look for an answer choice that overcomes that weakness. At the very least, your answer must address the issue of fairness — leaving you to consider only answer choices (A), (D), and (E).

Answer choice (A): This answer choice is at once too broad and too narrow. Even if the fairest economic system is one in which the values of most services are determined by the free market, it does not follow that a particularly high salary is fair — this only ensures the fairness of the system that allows it to exist. This answer choice strengthens the argument but does not provide sufficient support to prove the conclusion.

Answer choice (B): Even though this answer choice provides additional reasons for paying professional athletes more for their services, this is still an economic argument and not an ethical one — even if such salaries are necessary, that does not mean that they are fair. This answer choice is incorrect.

Answer choice (C): This answer choice might explain why teams' owners are forced to pay professional athletes high salaries, but it fails to prove that such salaries are fair. This answer choice is incorrect.

Answer choice (D): This is the correct answer choice. If any salary that a team owner is willing to pay is a fair salary, the conclusion follows logically from the rest of the argument. See discussion above.

Answer choice (E): Because it connects employers' willingness to pay a given salary to that salary's inherent fairness, answer choice (E) may seem attractive at first. However, notice that this answer choice does not prove that a professional athlete's salary is fair; rather, it presumes that it is, and then provides the rationale for its determination as a condition necessary for its fairness. Answer choice (E) is the Mistaken Reversal of what we need, and is therefore incorrect.
 mpoulson
  • Posts: 148
  • Joined: Mar 25, 2016
|
#23023
Hello,

I don't understand how to justify the conclusion in question 34 with the answer choices given. Answer D seems to make the premises about the unrestricted free market and profits for their team owner's irrelevant. If any salary is fair, then that would seem to indicate that the salary it is unrelated to profits or the market. I thought to justify a conclusion it would be wiser to incorporate support that discusses these points. Nevertheless, I chose E because I couldn't find anything more appealing. My question is why is D much better than E. Please clarify in simple terms. Appreciate the help and thank you.

- Micah
 Robert Carroll
PowerScore Staff
  • PowerScore Staff
  • Posts: 1787
  • Joined: Dec 06, 2013
|
#23079
Micah,

For a Justify the Conclusion question, you needn't worry about whether an answer choices fails to incorporate information in the stimulus, as long as it incorporates enough information to prove the conclusion is true. If the answer choice, when added to all the information in the stimulus, makes the conclusion definitely true, that answer choice is correct - no matter what. Thus, you should not avoid an answer choice simply because it keeps certain stimulus information irrelevant to the argument.

Answer choice (D) essentially says this:

team owner willing to pay it :arrow: salary fair

Because the conclusion of this argument is that the athletes' salaries are fair, if the premises contained information to prove the sufficient condition of this conditional, we could infer the necessary condition. And, indeed, the owners willingly pay these salaries, so, with the conditional in answer choice (D) added to the argument, we can properly infer the conclusion of the argument.

Answer choice (E) is also a conditional:

salary fair :arrow: salary determined by what an individual is willing to pay

Because the conclusion information is what the sufficient condition of this conditional says, this conditional is useful only if we already know the conclusion is true. But the point of this Justify the Conclusion question is that we don't know the conclusion is true even if the premises are as the author says. So the conditional in answer choice (E) does not get us from known information to unknown information - it's in the wrong order.

Robert Carroll
 lolaSur
  • Posts: 46
  • Joined: Nov 11, 2019
|
#72399
Hi!

I read your explanation and I went back to the stimulus, but I still can't wrap my head around why the correct diagraming is Team owner willing to pay :arrow: Salary fair

When I read the stimulus I understand that the salary is fair because team owners are willing to pay . So in my mind I am rationalizing this as if I know that if I see a salary that is fair I understand someone was willing to pay for that salary. What am I not understanding correctly?

I was stuck between choices D and E and chose E. In addition to my explanation above about how I thought about this argument, I chose E because the answer explicitly mentions unrestricted free markets, so I considered this answer to be more specific.

(For my reference: L5 A/J/S q34)
 Rachael Wilkenfeld
PowerScore Staff
  • PowerScore Staff
  • Posts: 1358
  • Joined: Dec 15, 2011
|
#72413
Hi Lolasur,

We want the conclusion to be the necessary part of the conditional, because that is the part we are trying to support. We want to link the premise and conclusion in a way that forces the conclusion to be true. In order to do that, we can look for a conditional relationship where the premise is sufficient to prove the conclusion.

In this stimulus, we link the premise (employers willing to pay) to the conclusion (fairness) via a conditional in answer choice (D). If the employer is willing to pay, then it is fair. That connects the premise from the stimulus to the conclusion in the stimulus in a way that proves the conclusion follows.

We don't want to do the opposite (if it's fair, then the employer is willing to pay) because the fairness was the part that wasn't established. That's what we need to prove with our answer choice.

Hope that helps!
Rachael
User avatar
 ellher
  • Posts: 1
  • Joined: Jul 26, 2021
|
#89067
It seemed to me that D was asking us to assume that the author believes that ANY salary is fair and thus would believe that a salary < min. wage would still be fair. Since the author discusses that the abnormally high salaries are fair and never mentions anything about abnormally low salaries, I chose A, despite not quite liking the general language it used.

Is this one of those LSAT situations where we abandon our real-life notions (that athletes compete in a market that has a minimum wage) and believe that the market is truly unrestricted, ie. has no minimum wage?
User avatar
 atierney
PowerScore Staff
  • PowerScore Staff
  • Posts: 215
  • Joined: Jul 06, 2021
|
#89146
Hi Ellher,

In answer to your second point, you generally want to check your real world notions at the door of the LSAT; certainly there will be some questions where real world context might assist you in some way; however, with any argument your usually evaluating its construction based on premises that are taken to be true (never fight the premises on the LSAT!). So here, we are considering what has been explicitly stated within the argument, that the athletes compete in an unrestricted free market, that in such a market, the salaries are determined by how much someone is willing to pay them, and that the salaries the athletes receive are based on what the owners are willing to fork out for the athlete's services. Now, the conclusion that said salary is fair is a bit of a jump. We must connect fairness to willingness of the owners to pay the athletes in order to validate the argument, and that is precisely what answer choice D does.

As to your first point on what D is asking us to assume, I would agree with you on some level that answer choice D proposes stating a salary of say below minimum wage being fair IF that is all the owners of the league were willing to pay. However, your assessment that this is not fair (which I think makes some sense) is only based on your real world notions of fairness as you stated, not the fact that the idea is to have a free market system wherein the services rendered are based on supply and demand for said services. In such a system of course, we wouldn't athletes to command salaries of lower than minimum wage. I certainly couldn't do what they do (but you know, maybe that goes both way as well. ;).

Let me know if you have further questions.
User avatar
 owen.don
  • Posts: 1
  • Joined: Mar 03, 2024
|
#105513
If we weren't able to recognize the logical gap between an employer's willingness to pay a high salary and that salary's inherent fairness, how would one go about improving this ability to recognize such things when reading LR stimulus?
User avatar
 Dana D
PowerScore Staff
  • PowerScore Staff
  • Posts: 117
  • Joined: Feb 06, 2024
|
#105528
Hey Owen,

Once you read a stimulus and identify an argument, you want to analyze whether it is a strong or good argument. You can do this by identifying the conclusion and then the premises or logic used to reach that conclusion. In this case, the stimulus breaks down as:

Premises: in an unrestricted free market, such as the market these athletes compete in, salaries are determined by what someone else is willing to pay for their services
Premises: athletes make enormous profits for their teams’ owners, and that is why owners are willing to pay them extraordinary salaries.
Conclusion: Professional athletes receive salaries that are fair.

Looking at the conclusion, do the 2 premises support it in such a way that there are no gaps in the logic or are there ways to challenge this conclusion? One thing to look for when considering if there are gaps in the argument are when there are concepts used in the conclusion that are not introduced or supported by the premises - in this case, the conclusion has to do with what is fair, while the premises never address the concept of fairness at all, they only explain why people are willing to pay athletes that much.

Is someone being willing to pay an athlete a lot of money the same thing as it being fair that athletes get paid a lot of money? We need to tie the notion of fairness to the premises in order to strengthen this argument, otherwise we're leaving room for someone to come in and point out this gaping disconnect.

Get the most out of your LSAT Prep Plus subscription.

Analyze and track your performance with our Testing and Analytics Package.